Set theory cartesian products and difference












0














First of all hello to everyone!



I have been stuck on the same exercise and I'm looking for proper guidance in order to solve this task.
I have $Atimes(Bsetminus C)=(Atimes B) setminus (Atimes C)$ ( sorry for not using the proper math format on this page, I'm new so I don't know how :D), and the problem is not proving the right side but the left.($Atimes (Bsetminus C)$).



I get to the point where ($xin A$ and $yin B$) and ($xin A$ and $ynotin C$) and don't know what to do?



Can someone please explain ?










share|cite|improve this question




















  • 1




    what do you mean by proving the left side? You are supposed to prove an equality of sets so you have to show left is a subset of right and right is a subset of left.
    – Anurag A
    Nov 24 at 19:27
















0














First of all hello to everyone!



I have been stuck on the same exercise and I'm looking for proper guidance in order to solve this task.
I have $Atimes(Bsetminus C)=(Atimes B) setminus (Atimes C)$ ( sorry for not using the proper math format on this page, I'm new so I don't know how :D), and the problem is not proving the right side but the left.($Atimes (Bsetminus C)$).



I get to the point where ($xin A$ and $yin B$) and ($xin A$ and $ynotin C$) and don't know what to do?



Can someone please explain ?










share|cite|improve this question




















  • 1




    what do you mean by proving the left side? You are supposed to prove an equality of sets so you have to show left is a subset of right and right is a subset of left.
    – Anurag A
    Nov 24 at 19:27














0












0








0







First of all hello to everyone!



I have been stuck on the same exercise and I'm looking for proper guidance in order to solve this task.
I have $Atimes(Bsetminus C)=(Atimes B) setminus (Atimes C)$ ( sorry for not using the proper math format on this page, I'm new so I don't know how :D), and the problem is not proving the right side but the left.($Atimes (Bsetminus C)$).



I get to the point where ($xin A$ and $yin B$) and ($xin A$ and $ynotin C$) and don't know what to do?



Can someone please explain ?










share|cite|improve this question















First of all hello to everyone!



I have been stuck on the same exercise and I'm looking for proper guidance in order to solve this task.
I have $Atimes(Bsetminus C)=(Atimes B) setminus (Atimes C)$ ( sorry for not using the proper math format on this page, I'm new so I don't know how :D), and the problem is not proving the right side but the left.($Atimes (Bsetminus C)$).



I get to the point where ($xin A$ and $yin B$) and ($xin A$ and $ynotin C$) and don't know what to do?



Can someone please explain ?







elementary-set-theory






share|cite|improve this question















share|cite|improve this question













share|cite|improve this question




share|cite|improve this question








edited Nov 24 at 19:24









MPW

29.8k11956




29.8k11956










asked Nov 24 at 19:20









PotentialMemory

1




1








  • 1




    what do you mean by proving the left side? You are supposed to prove an equality of sets so you have to show left is a subset of right and right is a subset of left.
    – Anurag A
    Nov 24 at 19:27














  • 1




    what do you mean by proving the left side? You are supposed to prove an equality of sets so you have to show left is a subset of right and right is a subset of left.
    – Anurag A
    Nov 24 at 19:27








1




1




what do you mean by proving the left side? You are supposed to prove an equality of sets so you have to show left is a subset of right and right is a subset of left.
– Anurag A
Nov 24 at 19:27




what do you mean by proving the left side? You are supposed to prove an equality of sets so you have to show left is a subset of right and right is a subset of left.
– Anurag A
Nov 24 at 19:27










1 Answer
1






active

oldest

votes


















0














If I understand, you are trying to prove membership in RHS implies membership in LHS.



You should then start by assuming $(x,y)in Atimes B$ and $(x,y)notin Atimes C$, since that's what it means to have $(x,y)in (Atimes B)setminus (Atimes C)$.



Your goal is to prove that $xin A$ and $y in Bsetminus C$, since that's what it means to have $(x,y)in Atimes (Bsetminus C)$.



This implies that $xin A$, that $yin B$, and that $ynotin C$. Why? Because if $(x,y)notin Atimes C$, then either $xnotin A$ or $ynotin C$ -- but you already know that $xin A$, so it must be the case that $ynotin C$.



You should be able to proceed from here.





For the other direction, start by assuming that $xin A$ and $y in Bsetminus C$, since that's what it means to have $(x,y)in Atimes (Bsetminus C)$. Then $yin B$ and $ynotin C$, so surely "$xin A$ and $yin B$" is true, so $(x,y)in Atimes B$. You should be able to finish the proof from here. Can you see that it is also true that $(x,y)notin Atimes C$?






share|cite|improve this answer























  • I have actually done that I'm trying to prove the LHS and that's where I'm having troubles.
    – PotentialMemory
    Nov 24 at 19:34










  • Okay, then I misunderstood. But the proof is completely reversible. Try it!
    – MPW
    Nov 24 at 19:38











Your Answer





StackExchange.ifUsing("editor", function () {
return StackExchange.using("mathjaxEditing", function () {
StackExchange.MarkdownEditor.creationCallbacks.add(function (editor, postfix) {
StackExchange.mathjaxEditing.prepareWmdForMathJax(editor, postfix, [["$", "$"], ["\\(","\\)"]]);
});
});
}, "mathjax-editing");

StackExchange.ready(function() {
var channelOptions = {
tags: "".split(" "),
id: "69"
};
initTagRenderer("".split(" "), "".split(" "), channelOptions);

StackExchange.using("externalEditor", function() {
// Have to fire editor after snippets, if snippets enabled
if (StackExchange.settings.snippets.snippetsEnabled) {
StackExchange.using("snippets", function() {
createEditor();
});
}
else {
createEditor();
}
});

function createEditor() {
StackExchange.prepareEditor({
heartbeatType: 'answer',
autoActivateHeartbeat: false,
convertImagesToLinks: true,
noModals: true,
showLowRepImageUploadWarning: true,
reputationToPostImages: 10,
bindNavPrevention: true,
postfix: "",
imageUploader: {
brandingHtml: "Powered by u003ca class="icon-imgur-white" href="https://imgur.com/"u003eu003c/au003e",
contentPolicyHtml: "User contributions licensed under u003ca href="https://creativecommons.org/licenses/by-sa/3.0/"u003ecc by-sa 3.0 with attribution requiredu003c/au003e u003ca href="https://stackoverflow.com/legal/content-policy"u003e(content policy)u003c/au003e",
allowUrls: true
},
noCode: true, onDemand: true,
discardSelector: ".discard-answer"
,immediatelyShowMarkdownHelp:true
});


}
});














draft saved

draft discarded


















StackExchange.ready(
function () {
StackExchange.openid.initPostLogin('.new-post-login', 'https%3a%2f%2fmath.stackexchange.com%2fquestions%2f3011969%2fset-theory-cartesian-products-and-difference%23new-answer', 'question_page');
}
);

Post as a guest















Required, but never shown

























1 Answer
1






active

oldest

votes








1 Answer
1






active

oldest

votes









active

oldest

votes






active

oldest

votes









0














If I understand, you are trying to prove membership in RHS implies membership in LHS.



You should then start by assuming $(x,y)in Atimes B$ and $(x,y)notin Atimes C$, since that's what it means to have $(x,y)in (Atimes B)setminus (Atimes C)$.



Your goal is to prove that $xin A$ and $y in Bsetminus C$, since that's what it means to have $(x,y)in Atimes (Bsetminus C)$.



This implies that $xin A$, that $yin B$, and that $ynotin C$. Why? Because if $(x,y)notin Atimes C$, then either $xnotin A$ or $ynotin C$ -- but you already know that $xin A$, so it must be the case that $ynotin C$.



You should be able to proceed from here.





For the other direction, start by assuming that $xin A$ and $y in Bsetminus C$, since that's what it means to have $(x,y)in Atimes (Bsetminus C)$. Then $yin B$ and $ynotin C$, so surely "$xin A$ and $yin B$" is true, so $(x,y)in Atimes B$. You should be able to finish the proof from here. Can you see that it is also true that $(x,y)notin Atimes C$?






share|cite|improve this answer























  • I have actually done that I'm trying to prove the LHS and that's where I'm having troubles.
    – PotentialMemory
    Nov 24 at 19:34










  • Okay, then I misunderstood. But the proof is completely reversible. Try it!
    – MPW
    Nov 24 at 19:38
















0














If I understand, you are trying to prove membership in RHS implies membership in LHS.



You should then start by assuming $(x,y)in Atimes B$ and $(x,y)notin Atimes C$, since that's what it means to have $(x,y)in (Atimes B)setminus (Atimes C)$.



Your goal is to prove that $xin A$ and $y in Bsetminus C$, since that's what it means to have $(x,y)in Atimes (Bsetminus C)$.



This implies that $xin A$, that $yin B$, and that $ynotin C$. Why? Because if $(x,y)notin Atimes C$, then either $xnotin A$ or $ynotin C$ -- but you already know that $xin A$, so it must be the case that $ynotin C$.



You should be able to proceed from here.





For the other direction, start by assuming that $xin A$ and $y in Bsetminus C$, since that's what it means to have $(x,y)in Atimes (Bsetminus C)$. Then $yin B$ and $ynotin C$, so surely "$xin A$ and $yin B$" is true, so $(x,y)in Atimes B$. You should be able to finish the proof from here. Can you see that it is also true that $(x,y)notin Atimes C$?






share|cite|improve this answer























  • I have actually done that I'm trying to prove the LHS and that's where I'm having troubles.
    – PotentialMemory
    Nov 24 at 19:34










  • Okay, then I misunderstood. But the proof is completely reversible. Try it!
    – MPW
    Nov 24 at 19:38














0












0








0






If I understand, you are trying to prove membership in RHS implies membership in LHS.



You should then start by assuming $(x,y)in Atimes B$ and $(x,y)notin Atimes C$, since that's what it means to have $(x,y)in (Atimes B)setminus (Atimes C)$.



Your goal is to prove that $xin A$ and $y in Bsetminus C$, since that's what it means to have $(x,y)in Atimes (Bsetminus C)$.



This implies that $xin A$, that $yin B$, and that $ynotin C$. Why? Because if $(x,y)notin Atimes C$, then either $xnotin A$ or $ynotin C$ -- but you already know that $xin A$, so it must be the case that $ynotin C$.



You should be able to proceed from here.





For the other direction, start by assuming that $xin A$ and $y in Bsetminus C$, since that's what it means to have $(x,y)in Atimes (Bsetminus C)$. Then $yin B$ and $ynotin C$, so surely "$xin A$ and $yin B$" is true, so $(x,y)in Atimes B$. You should be able to finish the proof from here. Can you see that it is also true that $(x,y)notin Atimes C$?






share|cite|improve this answer














If I understand, you are trying to prove membership in RHS implies membership in LHS.



You should then start by assuming $(x,y)in Atimes B$ and $(x,y)notin Atimes C$, since that's what it means to have $(x,y)in (Atimes B)setminus (Atimes C)$.



Your goal is to prove that $xin A$ and $y in Bsetminus C$, since that's what it means to have $(x,y)in Atimes (Bsetminus C)$.



This implies that $xin A$, that $yin B$, and that $ynotin C$. Why? Because if $(x,y)notin Atimes C$, then either $xnotin A$ or $ynotin C$ -- but you already know that $xin A$, so it must be the case that $ynotin C$.



You should be able to proceed from here.





For the other direction, start by assuming that $xin A$ and $y in Bsetminus C$, since that's what it means to have $(x,y)in Atimes (Bsetminus C)$. Then $yin B$ and $ynotin C$, so surely "$xin A$ and $yin B$" is true, so $(x,y)in Atimes B$. You should be able to finish the proof from here. Can you see that it is also true that $(x,y)notin Atimes C$?







share|cite|improve this answer














share|cite|improve this answer



share|cite|improve this answer








edited Nov 24 at 19:44

























answered Nov 24 at 19:30









MPW

29.8k11956




29.8k11956












  • I have actually done that I'm trying to prove the LHS and that's where I'm having troubles.
    – PotentialMemory
    Nov 24 at 19:34










  • Okay, then I misunderstood. But the proof is completely reversible. Try it!
    – MPW
    Nov 24 at 19:38


















  • I have actually done that I'm trying to prove the LHS and that's where I'm having troubles.
    – PotentialMemory
    Nov 24 at 19:34










  • Okay, then I misunderstood. But the proof is completely reversible. Try it!
    – MPW
    Nov 24 at 19:38
















I have actually done that I'm trying to prove the LHS and that's where I'm having troubles.
– PotentialMemory
Nov 24 at 19:34




I have actually done that I'm trying to prove the LHS and that's where I'm having troubles.
– PotentialMemory
Nov 24 at 19:34












Okay, then I misunderstood. But the proof is completely reversible. Try it!
– MPW
Nov 24 at 19:38




Okay, then I misunderstood. But the proof is completely reversible. Try it!
– MPW
Nov 24 at 19:38


















draft saved

draft discarded




















































Thanks for contributing an answer to Mathematics Stack Exchange!


  • Please be sure to answer the question. Provide details and share your research!

But avoid



  • Asking for help, clarification, or responding to other answers.

  • Making statements based on opinion; back them up with references or personal experience.


Use MathJax to format equations. MathJax reference.


To learn more, see our tips on writing great answers.





Some of your past answers have not been well-received, and you're in danger of being blocked from answering.


Please pay close attention to the following guidance:


  • Please be sure to answer the question. Provide details and share your research!

But avoid



  • Asking for help, clarification, or responding to other answers.

  • Making statements based on opinion; back them up with references or personal experience.


To learn more, see our tips on writing great answers.




draft saved


draft discarded














StackExchange.ready(
function () {
StackExchange.openid.initPostLogin('.new-post-login', 'https%3a%2f%2fmath.stackexchange.com%2fquestions%2f3011969%2fset-theory-cartesian-products-and-difference%23new-answer', 'question_page');
}
);

Post as a guest















Required, but never shown





















































Required, but never shown














Required, but never shown












Required, but never shown







Required, but never shown

































Required, but never shown














Required, but never shown












Required, but never shown







Required, but never shown







Popular posts from this blog

Quarter-circle Tiles

build a pushdown automaton that recognizes the reverse language of a given pushdown automaton?

Mont Emei